전하와 자하를 동시에 두면 이로부터 만들어지는 전자기장이 각운동량을 갖는다는 사실은 잘 알려져 있다. 처음으로 이 계산을 한 것이 톰슨이었다던가. 이 계산은 각운동량의 양자화로부터 전하와 자하의 양자화를 유도해내는 과정인 Dirac quantisation 혹은 Dirac-Schwinger-Zwanziger quantisation을 정당화하는데 이용되기도 한다.


여튼, 정석적인 계산방법은 전하를 원점에, 자하를 적당한 z축상의 한 점에 둔 뒤 원통좌표계를 써서 각운동량을 계산하는 것인데 이 방법 말고 벡터미적분학을 적절히 이용해서 쉽게(?) 계산하는 방법이 있다. 이 방법이 있다는 것은 알고 있었는데 정확한 과정을 떠올리는데 만 하루가 걸리고 나니 조금 슬프지만.


먼저 전하를 원점에, 자하를 $\vec{r'}$에 두자. 그리고 다음과 같이 벡터 $\vec{\rho} := \vec{r} - \vec{r'}$를 정의한다. 전하와 자하가 만들어내는 전자기장은 다음과 같이 계산할 수 있다.

\[ \vec{J} = \int \vec{r} \times \vec{P} = \int \vec{r} \times \left( \vec{E} \times \vec{B} \right)  \]


전기장과 자기장을 쓰기 위한 단위계는 cgs를 택하기로 한다.

\[ \vec{E} = \frac{e \vec{r}}{r^3} \] \[ \vec{B} = \frac{g \vec{\rho}}{\rho^3} \]


실제 계산에 문제가 되는 항은 다음 항이다.

\[ \frac{\vec{r} \times ( \vec{r} \times \vec{\rho})}{r^3 \rho^3} \]


벡터 삼중곱을 쓰면 이 항은 다음과 같이 쉽게 정리할 수 있다.

\[ \frac{\vec{r} \times ( \vec{r} \times \vec{\rho})}{r^3 \rho^3} = \vec{r} \frac{ \vec{r} \cdot \vec{\rho}}{r^3 \rho^3} - \frac{\vec{\rho}}{r \rho^3} \]


이제부터 벡터미적분학의 묘미가 시작된다. 다음 등식은 어렵지 않게 증명 가능하다.

\[ (\nabla \phi) \cdot (\nabla \varphi) = \nabla \cdot (\phi \nabla \varphi) - \phi \nabla^2 \varphi \]


이 식을 $\vec{a}/a^3$꼴의 식에 적용한다.

\[ \frac{ \vec{r} \cdot \vec{\rho}}{r^3 \rho^3} = \nabla \frac{1}{r} \cdot \nabla \frac{1}{\rho} = \nabla \cdot \left( \frac{1}{r} \nabla \frac{1}{\rho} \right) - \frac{1}{r} \nabla^2 \frac{1}{\rho} \]


다음 항등식은 전자기학을 공부했으면 심심찮게 만날 수 있다.

\[ \nabla^2 \frac{1}{r} = - 4 \pi \delta^3 (\vec{r}) \]


정리하면

\[ \frac{\vec{r} \times ( \vec{r} \times \vec{\rho})}{r^3 \rho^3} = 4 \pi \frac{\vec{r}}{r} \delta^3 (\vec{\rho}) + \vec{r} \nabla \cdot \left( \frac{1}{r} \nabla \frac{1}{\rho} \right) + \frac{1}{r} \nabla \frac{1}{\rho} \]


또는, Einstein summation convention을 도입할 경우,

\[ \frac{\vec{r} \times ( \vec{r} \times \vec{\rho})}{r^3 \rho^3} = 4 \pi \frac{\vec{r}}{r} \delta^3 (\vec{\rho}) + \nabla_j \left( \frac{\vec{r}_i}{r} \nabla_j \frac{1}{\rho} \right) \]


가 되어 total divergence만 남는 것을 확인할 수 있다. 따라서,

\[ \vec{J} = e g \int 4 \pi \frac{\vec{r}}{r} \delta^3 (\vec{\rho}) + \nabla_j \left( \frac{\vec{r}_i}{r} \nabla_j \frac{1}{\rho} \right) = 4 \pi e g \hat{r'} + \oint \text{boundary terms} \]


으로 정리할 수 있으며, 약간의 order of magnitude analysis를 통해 boundary term은 0이 된다는 것을 증명하면 정리는 끝난다. 해당 증명은 어렵지 않으니 생략.

\[ \therefore \vec{J} = 4 \pi e g \hat{r'} \]


단위계가 엉망인데 계산과정이 중요한 것일 뿐이니 적당히 알아서 집어넣으시길...

Posted by 덱스터

블로그 이미지
A theorist takes on the world
덱스터
Yesterday
Today
Total

달력

 « |  » 2024.3
1 2
3 4 5 6 7 8 9
10 11 12 13 14 15 16
17 18 19 20 21 22 23
24 25 26 27 28 29 30
31

최근에 올라온 글

최근에 달린 댓글

글 보관함